Graph, figure and maximum and minimum

Click For Summary
The function f(x,y) = x^2*y is analyzed within the specified region defined by -1 ≤ x ≤ 1 and x ≤ y ≤ x+2. The minimum value occurs at f(-1,-1) = -1, while the maximum is at f(1,3) = 3. To find these extrema, one must evaluate the gradient of f within the region and examine the boundaries, which include specific lines and corner points. The boundaries are defined by x = -1, y = x, y = x + 2, and x = 1, with critical points identified at (-1, -1), (-1, 1), (1, 1), and (1, 3). Ultimately, checking the function values at these points confirms the maximum and minimum values.
Niles
Messages
1,834
Reaction score
0

Homework Statement


Please take a look at the attached figure.

It is the function f(x,y) = x^2*y in the interval -1 =< x =< 1 and x =< y =< x+2.

From the figure, is it correct that the minimum is in f(-1,-1) = -1 and the maximum is in f(1,3) = 3?
 

Attachments

  • graph.gif
    graph.gif
    30.3 KB · Views: 505
Physics news on Phys.org
The level curve is plottet as well
 
Do I have to give 2 or 3 coordinates?
 
Since f is a function of 2 variables, you specify the point at which f has a maximum or minimum by giving the x and y values of the point. The third variable shown in the graph is the value of f. In order to determine the maximum and minimum values, you can take the gradient of f inside the region shown and determine if there are any (x,y) points where it is 0. Then look on the boundaries. Since the problem says, " in the interval -1 =< x =< 1 and x =< y =< x+2" (that's not actually an interval, by the way. It should say "region".) The boundaries are x= -1, with y between -1 and 1 (replace x in the formula by -1 to get a function of y only), y= x with x between -1 and 1 (replace y in the formula by x to get a function of x only), y= x+2 with x between -1 and 1 (replace y in the formula by x+ 2 to get a function of x only), and x= 1 with y between 1 and 3 (replace x in the formula by 1 to get a function of x only). Again, take the derivative to find if there are any places where it is 0. Finally, the boundaries of those- the corners of the region, (-1, -1), (-1, 1), (1, 1), and (1, 3). Check the value of f at all of those points to see which gives the maximum value and which gives the minimum value.
 
Question: A clock's minute hand has length 4 and its hour hand has length 3. What is the distance between the tips at the moment when it is increasing most rapidly?(Putnam Exam Question) Answer: Making assumption that both the hands moves at constant angular velocities, the answer is ## \sqrt{7} .## But don't you think this assumption is somewhat doubtful and wrong?

Similar threads

  • · Replies 8 ·
Replies
8
Views
2K
  • · Replies 11 ·
Replies
11
Views
2K
  • · Replies 4 ·
Replies
4
Views
2K
  • · Replies 6 ·
Replies
6
Views
1K
  • · Replies 10 ·
Replies
10
Views
3K
  • · Replies 4 ·
Replies
4
Views
2K
Replies
3
Views
1K
  • · Replies 18 ·
Replies
18
Views
3K
  • · Replies 3 ·
Replies
3
Views
1K
Replies
30
Views
3K